find k so that x+2 is a factor of x^3 - kx^2 + 2x + 7k

Answers

Answer 1

Answer:

k =  4

Step-by-step explanation:

If (x + 2) is a factor then f(- 2) = 0

f(x) = x³ - kx² + 2x + 7k , then

f(- 2) = (- 2)³ - k(- 2)² + 2(- 2) + 7k = 0      

     ⇒   - 8 - k(4) - 4 + 7k = 0

     ⇒  - 8 - 4k - 4 + 7k = 0

     ⇒ 3k - 12 = 0 ( add 12 to both sides )

     ⇒ 3k = 12 (divide both sides by 3 )

     ⇒ k = 4

   

Answer 2

Answer:

k = 4

Step-by-step explanation:

If x + 2 is a factor of x³ - kx² + 2x + 7k then

the value of x = -2

Solve for k

f ( x ) = x³ - kx² + 2x + 7k

plug -2 as x in the expression.

f ( -2) = ( -2)³ - k ( -2)² + 2 ( -2 ) + 7 k = 0

expand the exponents

= -8 -4k -4 + 7k = 0

combine like terms

= -8 -4 + -4k + 7k = 0= -12 + 3k = 0

add 12 to both side

-12 + 12 + 3k = 123k = 12

divide each side by 3

3k / 3 = 12/3k = 4

Related Questions

A cinema is doing a promotion to celebrate their 50th anniversary for 1 week. They give

away a free drink to every 98th customer, a free bag of popcorn to every 112th customer and

a free cinema ticket to every 224th customer. Which lucky customer will be the first to

receive all 3 items?​

Answers

Answer:

1,568 customer

Step-by-step explanation:

Find the lowest common multiple of 98, 112, and 224

98 = 98, 196, 294, 392, 490, 588, 686, 784, 882, 980, 1078, 1176, 1274, 1372, 1470, 1568, 1666

112 = 112, 224, 336, 448, 560, 672, 784, 896, 1008, 1120, 1232, 1344, 1456, 1568, 1680, 1792, 1904

224 = 224, 448, 672, 896, 1120, 1344, 1568, 1792, 2016, 2240

The lowest common multiple of 98, 112, and 224 is 1568

Therefore, the 1,568th customer will be the first to receive all 3 iitem

What’s the equation of the line that passes through the point (-4,4) and has a slope of 3/4

Answers

Answer:

y-y1=m(x-x1)

y-4=3/4(x+4)

y=3/4x+7

Which of the following is the parent function of all absolute value functions?
f(x) = 3x
f(x) = |x|
f(x) = 2|x|
f(x)=x²

Answers

f(x)=|x|

a parent function is the basic function before it has undergone transformations.

The expression 2x and x² have the same value for only two values of x. What are these values?​

Answers

Answer:

0 and 2

Step-by-step explanation:

solve in attachment .​

Answers

Answer:

2 ( Option A )

Step-by-step explanation:

The given integral to us is ,

[tex]\longrightarrow \displaystyle \int_0^1 5x \sqrt{x}\ dx [/tex]

Here 5 is a constant so it can come out . So that,

[tex]\longrightarrow \displaystyle I = 5 \int_0^1 x \sqrt{x}\ dx [/tex]

Now we can write √x as ,

[tex]\longrightarrow I = \displaystyle 5 \int_0^1 x . x^{\frac{1}{2}} \ dx [/tex]

Simplify ,

[tex]\longrightarrow I = 5 \displaystyle \int_0^1 x^{\frac{3}{2}}\ dx [/tex]

By Power rule , the integral of x^3/2 wrt x is , 2/5x^5/2 . Therefore ,

[tex]\longrightarrow I = 5 \bigg( \dfrac{2}{5} x^{\frac{5}{2}} \bigg] ^1_0 \bigg) [/tex]

On simplifying we will get ,

[tex]\longrightarrow \underline{\underline{ I = 2 }}[/tex]

Step-by-step explanation:

[tex]thank \: you[/tex]

The product of integers a,b,c and d is 546 and if 1<a<b<c<d, what is the value of b+c?​

Answers

Hello,

546=2*3*7*13

a=2, b=3,c=7,d=13

b+c=3+7=10

find the domain of f(x)=sec(2x)

Answers

Answer:

*Refer the image attached

Step-by-step explanation:

*Refer the image attached

2 1/4 x 3 1/5 brainliest

Answers

Answer:

36/5

Step-by-step explanation:

9/4×16/5

144/20

36/5

hope this is helpful

Answer:

[tex]7\frac{1}{5}[/tex]

Step-by-step explanation:

1. start by turning the fractions improper fractions:

[tex]2\frac{1}{4} =\frac{9}{4}[/tex]

[tex]3\frac{1}{5} =\frac{16}{5}[/tex]

2. then multiply them together:

[tex]\frac{9}{4}[/tex] x [tex]\frac{16}{5}[/tex] = [tex]\frac{144}{20}[/tex]

3. then simplify the fraction:

[tex]\frac{144}{20}[/tex][tex]=\frac{36}{5}[/tex]

4. turn it into a proper fraction:

[tex]\frac{36}{5} =7\frac{1}{5}[/tex]

the area of a rectangular bathroom mirror is 20 square feet. it is 2 feet tall. how wide is it?

Answers

Area = length x width

Fill in the given information

20 = 2 x width

Solve for width by dividing both sides by 2

Width =20/2

Width = 10 feet

Answer:

It is 10 feet wide

Step-by-step explanation:

The area of a rectangle is:

A = Length x width

So if we have the area, finding the width means diving the area by the given length.

In this case, the area of the rectangle is 20 square feet and the length is 2 feet:

20/2 = 10

Therefore the missing width is 10 feet

pls help me don't know what to do

Answers

Answer:

x=15

Step-by-step explanation:

The 60 degree angle and the (x+45) degree angle are both the same degree because they are vertical angles.

So to solve, just subtract 45 from 60

60-45=15

That's your answer!

Hope this helps!

HELP SOMEONE PLEASE!!!!!!!!!!!!!!!!!!!

Answers

Answer:

3/10

Step-by-step explanation:

The question asks for the probability of finding 4 or more, so first, you must add up all of the teenagers that have 4 or more shoes. Both the 4 and 5 categories fit, so add 20 and 10. Then, find the total number of people in the sample size. Overall, 100 teenagers are represented. Finally, put the number of people that have 4 or more shoes, 30, over the sample size, 100. This equals 30/100 which should be simplified to 3/10.

Greg buys 60 garden plants at a cost price of $2.00 each to sell in his shop. He sells 25 of them at the profit of 75% and 18 of them at the profit of 35%. He sells the rest of the plants for 4/5 of the cost price calculate the profit or loss he makes from selling 60 plants stating if it is a profit or loss

Answers

Answer:

$43.30 profit

Step-by-step explanation:

Total cost of plant:

60*2 = 120

Greg makes total of:

25*(2 + 0.75*2) + 18*(2 + 0.35*2) + (60 - 25 - 18)*2*4/5 = 163.3

Since Greg mare than cost, he has a profit and the amount is:

163.3 - 120 = 43.3

According to the number line, what is the distance between points A and B?

0 6 units
7 units
O 12 units
O 14 units

Answers

Answer:

14 units

Step-by-step explanation:

A = - 2, B = 12

Therefore,

d(A, B) = 12 - (-2) = 12 + 2 = 14 units

A farmer builds a fence to enclose a rectangular pasture. He uses 155 feet of fence. Find the total area of the pasture if it is 45.5 feet long. The length is 6 and the height is 5 what is the width?

Answers

Answer: [tex]1456\ ft^2[/tex]

Step-by-step explanation:

Given

Length of whole fence is 155 feet

If the length of rectangle is 45.5 ft

Suppose width is w

Length of whole fence is perimeter which is given by

[tex]\Rightarrow 155=2(45.5+w)\\\Rightarrow 77.5=45.5+w\\\Rightarrow w=32\ ft[/tex]

Area of the rectangle is given by the product of length and width

[tex]\Rightarrow A=lw\\\Rightarrow A=45.5\times 32\\\Rightarrow A=1456\ ft^2[/tex]

Thus, total area of pasture is [tex]1456\ ft^2[/tex]

jordan wants to save to buy a car and decides to open a banking account that is offering an interest rate of 4.5% compounded annually how much will jordan have in the account after 5 years it he deposits $7,000 today?

Answers

Answer:

8,723.27$

Step-by-step explanation:

someone help me for this algebra task please

Answers

i just need to answer questions

Select the expression that represents the following statement: add 24 to the quotient of 16 and 8.

Answers

Answer:

16/8 + 24

Step-by-step explanation:

Find the H.C.F. of the following numbers using division method
(a) 24, 40

Answers

Answer:

Ok

Thank you for

your time

Cayden has several screws on a scale, and the scale reads 80.955 cayden add 1 more screw and the scale reads 84.81

Answers

Answer:

-3.855

Step-by-step explanation:

Not sure if you need the weight of each screw! but you just subtract 80.955-84.81

Answer:

1

Step-by-step explanation:

itnisbbsndnfnfnfnnfnfnfncjcjccjcj

A parabola opens upward. The parabola goes through the point (3,-1),
and the vertex is at (2,-2).

Find the value of A for the parabola. Show your work. Use Part 1 and 2 to write the equation of the parabola.

Answers

Answer:

a=1

Step-by-step explanation:

Hopefully this helps :)

The equation of the parabola is: y = (x - 2)² - 2. Finding the value of A

The vertex of the parabola is at (2,-2). Since the parabola opens upward, the equation of the parabola will be of the form:

y = A(x - 2)² - 2

We can plug the point (3,-1) into this equation to find the value of A.

-1 = A(3 - 2)² - 2

Simplifying the right side of the equation, we get:

-1 = A - 2

Adding 2 to both sides of the equation, we get:

1 = A

Therefore, the value of A is 1.

Writing the equation of the parabola

The equation of the parabola is:

y = (x - 2)² - 2

To know more about parabola:

https://brainly.com/question/11911877

#SPJ2

Hello, Brainly community!

This question is for all of those Calculus people out there.

The volume of a swimming pool is changing with respect to time, such that the volume is given by W(t), where W(t) is measured in cubic centimeters and t is measured in seconds. A tangent line is shown for W(t) at t = 3 seconds. Determine the best estimate for the value of the instantaneous rate of change of W(t) when t = 3.
(I've narrowed down the answer choices to 2, and just really need to find the right way of thinking to find the answer)

(A) W(lim t) as t goes to 3.
(B) [W(3.1) - W(2.9)] / 0.2.

Thank you in advance!

Answers

Answer:

(B)  [tex]\displaystyle \frac{W(3.1) - W(2.9)}{0.2}[/tex]

General Formulas and Concepts:

Calculus

Limits

Derivatives

The definition of a derivative is the slope of the tangent line.

Derivative Notation

Instantaneous Rates

Tangent Line: [tex]\displaystyle f'(x) = \frac{f(b) - f(a)}{b - a}[/tex]

Step-by-step explanation:

Since we are trying to find a rate at which W(t) changes, we must find the derivative at t = 3.

We are given 2 close answer choices that would have the same numerical answer but different meanings:

(A)  [tex]\displaystyle \lim_{t \to 3} W(t)[/tex](B)  [tex]\displaystyle \frac{W(3.1) - W(2.9)}{0.2}[/tex]

If we look at answer choice (A), we see that our units would simply just be volume. It would not have the units of a rate of change. Yes, it may be the closest numerically correct answer, but it does not tell us the rate at which the volume would be changing and it is not a derivative.

If we look at answer choice (B), we see that our units would be cm³/s, and that is most certainly a rate of change. Answer choice (B) is also a derivative at t = 3, and a derivative tells us what rate something is changing.

∴ Answer choice (B) will give us the best estimate for the value of the instantaneous rate of change of W(t) when t = 3.

Topic: AP Calculus AB/BC (Calculus I/I + II)

Unit: Differentiation

Book: College Calculus 10e

The formula for the circumference of a circle is R = c/2(pi)

Find the radius of a circle that has a circumference of 16(pi)

A) r = 4
B) r = 8
C) r = 12
D) r = 16

Answers

The answer is B) r= 8

On the unit circle, which of the following angles has the terminal point
coordinates.
A. 45
B. 135
C. 225
D. 315

Answers

Answer: C. 225

Step-by-step explanation:

What is -2y + -4y. Simplify the answer.

Answers

Step-by-step explanation:

Explanation is in the attachment

hope it is helpful to you

Answer:

[tex]-2y+\left(-4\right)y[/tex][tex]=-2y-4y[/tex][tex]=-6y[/tex]

[tex]-----------[/tex]

hope it helps...

have a great day!!

PLS HELP SOON WILL MARK BRAINLYEST

A railroad tunnel is shaped like a semi-ellipse, as shown below. A semiellipse is shown on the coordinate plane with vertices on the x axis and one point of intersection with the positive y axis. The height of the tunnel at the center is 35 ft, and the vertical clearance must be 21 ft at a point 8 ft from the center. Find an equation for the ellipse.

Answers

According to the question

b= 35 and (8,21) lies on the ellipse

After calculation we get a= 10

equation for the ellipse.

[tex] \frac{ {x}^{2} }{100} + \frac{ {y}^{2} }{1225} = 1[/tex]

Find a unit vector u u in R 2 R2 such that u u is perpendicular to v . v. How many such vectors are there

Answers

Answer: hello some part of your question is missing

Let v=〈−2,5〉 in R^2,and let y=〈0,3,−2〉 in R^3.

Find a unit vector u in R^2 such that u is perpendicular to v. How many such vectors are there?

answer:

One(1) unit vector ( < 5/√29,  2 /√29 >  ) perpendicular to 〈−2,5〉

Step-by-step explanation:

let  

u = < x , y > ∈/R^2  be perpendicular to  v = < -2, 5 > ------ ( 1 )

hence :

-2x + 5y = 0

-2x = -5y

x = 5/2 y

back to equation 1

u = < 5/2y, y >

∴ || u || = y/2 √29

u   = < 5 /2 y * 2 / y√29 ,  y*2 / y√29 >

    = < 5/√29,  2 /√29 >  ( unit vector perpendicular to < -2, 5 > )

y = RootIndex 3 StartRoot x EndRoot. y = negative (0.4) RootIndex 3 StartRoot x minus 2 EndRoot
Which of the following describes the graph of the transformed function compared with the parent function? Select all that apply.

Answers

Answer:

- Reflected over the x-axis  

- Compressed by a factor of 0.4.

- Translated 2 units left

Step-by-step explanation:

Given

[tex]y = \sqrt[3]{x}[/tex]

[tex]y' = -(0.4)\sqrt[3]{x-2}[/tex]

Required

The transformation from y to y'

First, y is reflected over the x-axis.

The transformation rule is:

[tex](x,y) \to (x,-y)[/tex]

So, we have:

[tex]y = \sqrt[3]{x}[/tex] becomes

[tex]y' = -\sqrt[3]{x}[/tex]

Next, it was compressed by a scale factor of 0.4

The rule is:

[tex]y' = k * y[/tex]

Where k is the scale factor (i.e. k = 0.4)

So, we have:

[tex]y' = 0.4 * -\sqrt[3]{x}[/tex]

[tex]y' = -(0.4)\sqrt[3]{x}[/tex]

Lastly, the function is translated 2 units left;

The rule is:

[tex](x,y) \to (x-2,y)[/tex]

So, we have:

[tex]y' = -(0.4)\sqrt[3]{x - 2}[/tex]

Answers:

-reflected over the x-axis

-translated 2 units right

-compressed by a factor of 0.4

It took Sarah 4 days to write a paper, she wrote 12 pages on day 1, 15 pages on day 2 and 9 pages on day 3. If she wrote 12 pages per day , how many pages did she write on the fourth day ?

Answers

Answer:

Option A

Step-by-step explanation:

Sarah took 4 days to write a paper.

She wrote 12 pages per day, so total number of pages she wrote in 4 days = 12 × 4

= 48 pages

On day 1, she wrote number of pages = 12

On day 2, she wrote number of pages = 15

On day 3, she wrote number of pages = 9

On day 4, she wrote number of pages = P

She wrote total number pages in 4 days = 12 + 15 + 9 + P

                                                                    = 36 + P

Therefore, P + 36 = 48

P = 48 - 36

P = 12

She wrote 12 pages on day 4.

Option A is the answer.

Help me?
185/100 + 50% + 4%=?

Answers

Answer:

2.39

Step-by-step explanation:

Convert everything to a decimal.  

To do that with a fraction divide the numerator by the denominator.  

185 divided by 100 = 1.85

185/100 = 1.85

To convert a percent to a decimal divide the number by 100.

50 divided by 100 = 0.5

50/100 = 0.5

4 divided by 100 = 0.04

4/100 = 0.04

Now add:

1.85 + 0.5 + 0.04 = 2.39

Sketch the graph of each of the following quadratic functions: (a) f(x) = -2x² + 7x + 4 for -1 ≤ x ≤ 5.
Help me with this ques pleasee,i'll mark u as the brainliest!!​

Answers

Answer:

Please find attached the graph of the function created with MS Excel showing the relevant points required to draw an approximate graph of the function on a graph paper

Step-by-step explanation:

The given quadratic function is f(x) = -2·x² + 7·x + 4

The range of the input (x) values = -1 ≤ x ≤ 5

The coefficient of the quadratic is negative -2, the graph is n shape

The intercept form of the function is given as follows;

-2·x² + 7·x + 4 = -1 × (2·x² - 7·x - 4)

-1 × (2·x² - 7·x - 4) = -1 × (2·x² + x - 8·x - 4)

-1 × (2·x² + x - 8·x - 4) = -1 × (x · (2·x + 1) - 4·(2·x + 1))

∴ -1 × (x · (2·x + 1) - 4·(2·x + 1)) = -1 × (2·x + 1)·(x - 4)

∴ f(x) = -2·x² + 7·x + 4 = -1 × (2·x + 1)·(x - 4)

At the x-intercepts, (2·x + 1) = 0 or (x - 4) = 0, which gives;

x = -1/2 or x = 4

Therefore, the x-intercepts are (-1/2, 0), (4, 0)

The equation in vertex form is given as follows;

f(x) = -2·x² + 7·x + 4 = -2·(x² - 7·x/2 + 2)

By applying completing the squares method, to x² - 7·x/2 - 2, we get;

Where x² - 7·x/2 - 2

x² - 7·x/2 = 2

x² - 7·x/2 + (-7/4)² = 2 + (-7/4)² = 81/15

(x - 7/4)² = 81/16

∴ (x - 7/4)² - 81/16 = 0 = x² - 7·x/2 - 2

∴ x² - 7·x/2 - 2 = (x - 7/4)² - 81/16

-2·(x² - 7·x/2 + 2) = -2·((x - 7/4)² - 81/16) = -2·(x - 7/4)² + 81/8

The vertex = (7/4, 81/8)

When x = 0, we get;

f(0) = -2 × 0² + 7 × 0 + 4 = 4

The y-intercept = (0, 4)

The sketch of the function should pass through the x-intercepts (-1/2, 0), (4, 0), the y-intercept (0, 4), and the y-intercept (0, 4), and the vertex, (7/4, 81/8) on a graph sheet

Please find attached a drawing of the function of the function created with MS Excel

Other Questions
On a coordinate plane, rhombus W X Y Z is shown. Point W is at (7, 2), point X is at (5, negative 1), point Y is at (3, 2), and point Z is at (5, 5).What is the perimeter of rhombus WXYZ?StartRoot 13 EndRoot units12 unitsStartRoot 13 EndRoot units20 units Cross out three digits in the number 51489704 so that the resulting number is divisible by 45. What number is left? Hi can i please get help on this question Find the volume of this sphere.Round to the nearest tenth.16 ftFormulas for Spheres[? ] ftEnter $ 485,000 $ 432,000 $Enter a dollar amount Enter percentages rounded to 0 decimal places % Inventory $ 786,000 $ 617,000 $Enter a dollar amount Enter percentages rounded to 0 decimal places % Total assets $3,111,000 $2,707,000 $Enter a dollar amount Enter percentages rounded to 0 decimal places % Which expression is equivalent to 3 square root of x^5*y All functions have a domain and a range.TrueFalse Need an answer quick!!Lines A and B are parallelA123125B5 678m26 = [? ] A sample of 25 one-year-old girls had a mean weight of 24.1 pounds with a standard deviation of pounds. Assume that the population of weights is normally distributed. A pediatrician claims that the standard deviation of the weights of one-year-old girls is less than pounds. Do the data provide convincing evidence that the pediatrician's claim is true effects of drug in an individual, family and society SOMEONE HELP ASAP PLES NO EXPLANATOIN NEEDED PLS LEAVE UR ANSWER AS TEXT (SOME TIMES I CAN'T SEE IMAGES) THANK YOU SO MUCH!!! Complete the sentence with the pronoun that agrees with the antecedent. If students would like to be part of the class play,we should come to the drama room this afternoon.you should come to the drama room this afternoon.they should come to the drama room this afternoon.he or she should come to the drama room this afternoon. Please help! Which answer offers the best explanation for why Tecumseh fought alongside the British in the War of 1812? Tecumseh was well paid by British generals. Tecumseh wanted to preserve trade with Britain. Tecumseh considered the United States to be an enemy. Tecumseh felt his homeland was more secure with the United States. Morwenna, Inc. reports the following information for August: Sales Revenue $900,000 Variable Cost of Goods Sold 120,000 Fixed Cost of Goods Sold 60,000 Variable Selling and Administrative Costs 150,000 Fixed Selling and Administrative Costs 50,000 Calculate the operating income for August using absorption costing. When the E string of a guitar (frequency 330 Hz) is plucked, the sound intensity decreases by a factor of 2 after 4 s. Determine Each of the following factors affects the weighted average cost of capital (WACC) equation. Which are factors that a firm can control?A. Interest rates in the economy B. Tax rates C. The firm's dividend payout ratio D. The general level of stock prices A sealed vessel initially contains 100 g of chlorine gas and 90 g of hydrogen gas. The two gases undergo reaction to form HCl. Which of the following statements is true?a. 100 g HCl is producedb. 190 g HCl is producedc. less than 90 g HCl is producedd. between 100 and 190 g of HCl is produced Escoge la mejor respuesta, segn el contexto. Qu ___ en la television ahora? a. mires b. miraras c. ests mirando d. has mirado Arrange the following fact mention two institutions that can help to claim our right in Ghana If 260 people donated $10.00 to an organization how much money would said organization have received in total?